You are on page 1of 45

NATIONAL LAW TRAINING INSTITUTE

CLAT MOCK TEST SERIES


CLAT MOCK-4

Duration: 120 Minutes Name of the Candidate: ____________________

Maximum Marks: 150 Contact Number: _________________________

INSTRUCTIONS TO CANDIDATES

1. The test Paper is for Five Year integrated Law Programme for 150 marks
containing 150 multiple Choice Questions.
2. There will be Negative marking for multiple choice objective type questions.
0.25 marks will be deducted for every wrong answer or where candidates have
marked more than one response.
3. Use BLACK/BLUE BALL POINT PEN only for writing the roll No. and
other details on OMR response Sheet.
4. Indicate only the most appropriate answer by shading from the options
provided. the answer circle should be shaded completely without leaving any
space.
5. As the responses cannot be modified/corrected on the OMR Response Sheet,
candidates have to take necessary precautions before marking the appropriate
circle.

ALL THE BEST!!

ENROL NOW: https://www.clatnlti.com/


Join us on Telegram! https://t.me/lawNLTI

1 | NLTI CLAT MOCK-4 – 26TH FEBRUARY 2023


ENGLISH SECTION

It’s a rare speech where you go onstage hoping to get booed.

Look, folks — to borrow a much-repeated phrase from Tuesday night — I can’t know what was in President
Biden’s mind during the State of the Union address. I’m not sure if he or his advisers planned the speech
thinking that sparring with members of the new Republican House majority would make the opposition look
more unhinged or confer on him a useful image of feistiness and vigor.

But his adversaries obliged regardless, in a prime-time speech that began with optimism and comity, then
devolved into the most overt heckling we’ve ever seen at a State of the Union.

And I don’t think the president minded.

Reportedly, the new House speaker, Kevin McCarthy had urged his delegation to remember that cameras were
on them at all times, and to behave.

Their newfound discipline lasted minutes. Mr. Biden, who opened the speech with an upbeat report on the
economy and a list of achievements, got applause from both sides of the chamber for citing bipartisan
legislation.

But when Mr. Biden accused some Republicans of wanting to use the debt ceiling to force cuts to Social
Security and Medicare, GOP representatives hooted and howled. Representative Marjorie Taylor Greene of
Georgia — easily spotted in a white, fur-lined coat — called out, “Liar!”

When Representative Joe Wilson of South Carolina yelled “You lie!” at President Barack Obama at a 2009
address, it necessitated an apology and rebuke. Now it’s just the warm-up.

As Republicans continued to jeer, Mr. Biden seemed to enjoy ad-libbing responses: “Look it up!” When Mr.
Biden later addressed a father in the audience whose daughter had died from a fentanyl overdose, and a
lawmaker shouted, “It’s your fault!”

Mr. McCarthy was left to visibly shush his colleagues over and over from his seat behind the president. He
spent much of the speech glumly leaning back like an assistant principal at an unruly school assembly.

Mr. Biden came into the event with his own challenges and points to make. He is expected to decide soon
whether to run for a second term, and at age 80 he faces doubts even among his supporters about whether he’s
up to the demands of six more years in office, not to mention the campaign trail.

Of course, the State of the Union is about policy proposals and a report on the health of the country. As far as
the Constitution is concerned, an email could suffice. But as a speech watched by millions on TV, it is meant
to send messages not just about substantive goals but about the person at the podium.

2 | NLTI CLAT MOCK-4 – 26TH FEBRUARY 2023


This address seemed written and practiced with energy in mind. Mr. Biden delivered it fast and loud, punching
words like “unbowed and unbroken.” The sentences and phrases were tight; Mr. Biden, who has dealt with a
stutter, stumbled only occasionally. His speech had more “folks” than the Newport festival. And the back-
and-forth with hecklers gave it juice — not to mention, clips bound to be played and replayed on the next
day’s news.

Mr. Biden’s presidency has not exactly been known for galvanizing TV. Josh Tyrangiel, a television news
producer (and my former editor), recently suggested in The New York Times that Mr. Biden shake up his
address with a multimedia production, as the Jan. 6 committee so effectively did.
(https://www.nytimes.com/2023/02/08/arts/television/state-of-the-union-republican-heckling.html)

1. What did the back-and-forth with hecklers during the State of the Union address give to
President Biden's speech, according to the article?
a) Controversy and negative attention
b) A touch of energy and juice
c) A lack of credibility and authority
d) A sense of disunity and chaos

2. How did House Speaker Kevin McCarthy respond to his colleagues' heckling during the State of
the Union address?
a) He joined in on the heckling
b) He visibly shushed his colleagues from his seat
c) He ignored the heckling and focused on the speech
d) He left the chamber in protest

3. What was the initial response of both sides of the chamber when Mr. Biden cited bipartisan
legislation in his State of the Union address?
a) They remained silent
b) They cheered and applauded
c) They hooted and howled
d) They shouted "Liar!"

4. How did President Biden deliver his State of the Union address?
a) Slowly and softly
b) Fast and loud, with tight sentences and phrases
c) With a stutter and stumbling speech
d) Using multimedia production to keep viewers engaged

5. What was the actual purpose of Mr. Biden's State of the Union address?
a) To send messages about the person at the podium
b) To report on policy proposals
c) To provide a report on the health of the country
d) All of the above

Solitude is a state of being that has been both revered and feared throughout human history. It is a condition
that can bring forth immense personal growth and introspection, yet it can also evoke feelings of loneliness
3 | NLTI CLAT MOCK-4 – 26TH FEBRUARY 2023
and isolation. As a species, we are wired for social connection and yet, in moments of solitude, we are forced
to confront our own thoughts and feelings in a way that can be both exhilarating and overwhelming.

Solitude is often associated with monastic or ascetic lifestyles, where individuals retreat from society in order
to focus on their spiritual or intellectual pursuits. But even in the bustling metropolises of the modern world,
solitude can be found in the quiet moments between interactions, in the stillness of a museum gallery, or in
the pages of a book.

The experience of solitude is often subjective and varies from person to person. For some, solitude is a
welcome respite from the chaos and noise of the world, a time to recharge and reflect. For others, it can be an
intimidating and unpleasant experience, filled with fear, anxiety, and self-doubt. Regardless of our individual
experiences, however, solitude forces us to confront the reality of our existence and our place in the world.

In solitude, we are faced with our own thoughts and emotions, stripped of the distractions of daily life. It is a
time to reflect on our relationships, our choices, and our aspirations. For those who are able to embrace this
introspection, solitude can be a transformative experience, leading to a deeper understanding of oneself and a
greater appreciation for life.

Yet, despite its potential for personal growth and insight, solitude can also be a source of suffering. Loneliness,
a common experience for those who spend extended periods of time in solitude, can be a powerful force that
brings forth feelings of emptiness and despair. The absence of human connection and support can be
devastating, and those who suffer from chronic loneliness often struggle with depression and other mental
health issues.

Solitude, then, is a double-edged sword, one that can bring forth both enlightenment and suffering. It is a
powerful tool that can be harnessed for personal growth and self-discovery, but it must be approached with
caution and a deep understanding of its effects on the human psyche.

Solitude is a complex and multifaceted condition that reflects the human condition itself. It is a space where
we are free to explore our thoughts, emotions, and desires, but it is also a place where we are forced to confront
our own limitations and weaknesses. Whether we find solace in the quiet moments of solitude or suffer in its
isolation, it is an experience that demands our attention and demands that we examine the nature of our
existence.
(Source: Camarillo, Lauren A. Solitude: A Human Condition. Diss. 2018.)

6. According to the passage, what is the subjective experience of solitude?


a) It is a universally negative experience for all individuals.
b) It is a positive experience that brings forth feelings of enlightenment.
c) It is a personal experience that varies from person to person.
d) It is a social experience that involves interacting with like-minded individuals.

7. Which of the following options is an assumption being made in the passage?


a) Solitude is an experience that demands our attention and examination.
b) Solitude is a double-edged sword, one that can bring forth both enlightenment and suffering.
c) The experience of solitude is often subjective and varies from person to person.
d) None of the above
4 | NLTI CLAT MOCK-4 – 26TH FEBRUARY 2023
8. What is the main point of the passage?
a) That solitude is always a negative experience.
b) That human beings are not wired for social connection.
c) That solitude can have both positive and negative effects on an individual.
d) That solitude is an outdated concept in modern society.

9. How might the experience of solitude differ in a monastic or ascetic lifestyle versus in a modern
city?
a) In a monastic or ascetic lifestyle, solitude is forced upon the individual, while in a modern city it is a
choice.
b) In a modern city, solitude is associated with spiritual or intellectual pursuits, while in a monastic or
ascetic lifestyle it is associated with loneliness.
c) In a modern city, solitude is temporary and easily interrupted, while in a monastic or ascetic lifestyle
it is more prolonged and immersive.
d) In a monastic or ascetic lifestyle, solitude is typically associated with socializing with other like-
minded individuals, while in a modern city it is a more individualistic experience.

10. Which of the following best characterizes the author's attitude towards solitude in the passage?
a) The author is highly critical of those who embrace solitude as a means of spiritual or intellectual
pursuit, and views it as an outdated and impractical concept in modern society.
b) The author is highly skeptical of solitude and dismisses it as a frivolous and self-indulgent pursuit
that ultimately does more harm than good.
c) The author is ambivalent about solitude, recognizing its potential for both positive and negative
outcomes, but ultimately sees it as a personal choice that should be left up to the individual.
d) The author is generally positive towards solitude, viewing it as a valuable and necessary experience
for all individuals, regardless of their circumstances or personal preferences, but recognizes the
challenges that come with extended periods of isolation.

George and Lennie, two migrant workers, were walking down the dusty roads of California, on their way to
their next job. George was a small, wiry man with sharp features, while Lennie was a giant of a man with a
childlike innocence and a tendency to accidentally cause trouble. Despite their differences, the two men were
the best of friends and had a dream of one day owning a farm of their own, a place where Lennie could care
for the rabbits and George could finally escape the drudgery of life as a migrant worker.

They arrived at their new job on a sprawling ranch, where they were greeted by a group of men who were
wary of their presence. The ranch was filled with a cast of characters, each with their own stories and struggles.
There was Candy, an old swamper who was missing a hand and desperate to secure a place in the future farm
of George and Lennie; Slim, a mule driver who was respected by all the men for his skill and quiet demeanor;
and Curley, the boss's aggressive son, who was always on the lookout for a fight.

Despite the rough conditions and the hostile environment, George and Lennie managed to find a small corner
of the ranch where they could escape from the world. They talked about their dream and the future, imagining
a place where they could live without fear of being separated or mistreated. It was a place where Lennie could
be safe, where he wouldn't have to worry about breaking things or causing trouble.
5 | NLTI CLAT MOCK-4 – 26TH FEBRUARY 2023
However, their dream was soon shattered when Curley's wife entered the picture. Despite being warned by
the other men to stay away from her, Lennie became fascinated with her and wouldn't leave her alone. One
day, when she tried to speak to him, Lennie accidentally killed her in a fit of panic. The men on the ranch soon
discovered what had happened and decided that Lennie needed to be killed before he could cause any more
trouble.

George knew that he had to take matters into his own hands and protect his friend. He led Lennie to a secluded
spot near the river, where they sat and talked about their dream one last time. George told Lennie that he was
going to shoot him and make it look like an accident, so that he wouldn't suffer. In his final moments, Lennie
finally understood what was happening and whispered, "Tell George I was a good guy."

George pulled out his gun and shot Lennie, ending his life. He then returned to the ranch and told the men that
he had found Lennie dead by the river, just as they had planned. The men on the ranch were shocked and
saddened by the news, realizing the depth of the bond between George and Lennie and the tragedy of their
shattered dream.

In the end, George was left alone, wandering down the road once again, searching for a new purpose in life.
He knew that he would never be able to forget Lennie or their dream, but he also knew that it was time to
move on and find a new path. Despite the sadness of losing his friend, George found solace in the knowledge
that Lennie was finally free from a world that had never truly understood him.

(Source: Steinbeck, J. (1937). Of Mice and Men. Penguin Group.)

11. Which literary device is being used in the underlined sentence from the passage?
a) Litotes
b) Metaphor
c) Foreshadowing
d) Synecdoche

12. What can we infer from the passage?


a) George and Lennie had a perfect friendship and never had any problems.
b) Lennie's accidental killing of Curley's wife led to tragedy and the end of their dream.
c) Curley's wife was the only character on the ranch who understood Lennie.
d) George had no remorse for shooting Lennie and felt justified in his actions.

13. What was the reaction of George after he slayed his best friend?
a) He immediately left the ranch and never looked back as he was so consumed by guilt
b) He did not tell the workers that he shot Lennie and he was harrowed yet some sense of comfort was
there.
c) He took responsibility for Lennie's death and was confessed to his crime
d) He was indifferent to Lennie's death and showed no emotion to his coworkers yet he felt extremely
distraught going forward

14. Which of the following can be used instead of the word “SOLACE”?
a) Consolation
6 | NLTI CLAT MOCK-4 – 26TH FEBRUARY 2023
b) Enforce
c) Cavern
d) Regulate

15. Why did the men on the ranch distrust George and Lennie, and how did they treat them?
a) Poignant and elegiac
b) Farcical and absurd
c) Cynical and sardonic
d) Inscrutable and enigmatic

Gone are the days of cutting hearts from red construction paper and sending kids off with a shoebox to collect
equally rudimentary holiday cards from classmates. Class Valentines are now an expensive, elaborate and
competitive business. And the burden of obtaining valentines and making them just right, like most things
relating at all to parenthood, falls squarely upon moms.

The superpower of NYT reporter drama "She Said" is also its undoing: Motherhood
The unspoken — or often, actually and clearly spoken — rule of school is that you must bring something for
everyone in the class, or for no one. The first issue of class valentines is making sure no kid is left out, no
name spelled incorrectly, which becomes difficult when dealing with young spellers, readers and writers.
Writing out 30 classmates' names was a chore for my child, more homework than his actual homework at that
age.

But sometime between kindergarten and first or second grade, I noticed the valentines coming home were
much fancier than the ones I had sent off in his backpack (along with larger cards for the teacher, teacher's
aide, principal — and extras). Parents were giving away toys wrapped with cards, whole candy bars or cookies.
A card was no longer enough. A handmade card was embarrassing.

A search for class valentines on Amazon pulls up pages of cards with something else: cards with pencils, balls,
fidget toys, rubber duckies, and more — and if you're going the plain card route, your selection is much more
limited than if you wanted, say, finger puppet valentines or glow stick valentines or lip gloss valentines or
foam airplane kit valentines.

One of the aspects I never anticipated about parenthood was how much plastic comes into the house, how
many little tiny plastic crap things a child accumulates from well-meaning family, friends — or classmates on
Valentine's Day. These valentines (and then some) also add up financially. This year, spending for Valentine's
Day is expected to be almost $26 billion, making 2023 one of the highest spending years on record, according
to CNBC. On average, Americans will spend $192.80 on Valentine's Day this year, up from last year, as
CNBC reports, but their article on how to save money on Valentine's Day gives tips for couples only. Nothing
about school valentines.

Valentines, like the birthday treats of yore, have become a way to win friends and influence people, with
moms as campaign managers.
The pressure and the race is on. At stake?
Respect from your child's teacher — you didn't forget, you didn't leave anybody out — and most importantly,
social status for your kid. Because like every messed-up thing about school (and probably, parenthood),
Valentine's Day cards have become a competition.
7 | NLTI CLAT MOCK-4 – 26TH FEBRUARY 2023
Valentines, like the birthday treats of yore, have become a way to win friends and influence people, with
moms as campaign managers. Because the people shopping for perfect valentines, the people paying for them,
making sure every name is correct and sending their children off to school safely with all the heart-shaped
bribes are overwhelmingly women.

(Source: https://www.salon.com/2023/02/08/school-class-valentines-competitive-cost/)
16. What is the main focus of this article?
a) Valentines have become a competition in schools
b) Americans are expected to spend a lot of money on Valentine's Day
c) The role of mothers in obtaining class valentines
d) The burden of making class valentines falls on mothers

17. How does the trend of giving fancy valentines for children impact mothers?
a) It adds to the household plastic waste
b) It reduces the workload of mothers
c) It increases the workload and pressure on mothers
d) It improves the social status of mothers

18. According to the passage, what is the main issue with obtaining class valentines for children?
a) The competition between mothers
b) The difficulty in spelling classmates' names correctly
c) The limited selection of plain cardS
d) The increased spending on Valentine's Day

19. What is the biggest problem with making class valentines?


a) The author is neutral about the use of plastic
b) The author thinks that the use of plastic is positive
c) The author thinks that the use of plastic is unnecessary
d) The author thinks that the use of plastic is a problem

20. What is one reason why class valentines have become more elaborate and expensive?
a) The pressure to win friends and influence people
b) The high spending expected on Valentine's Day
c) The increase in the number of classmates
d) The lack of options for plain cards

Legislators want to make abortion illegal as soon as an embryo has detectable cardiac activity. These so-called
“fetal heartbeat laws” and their underlying talking point that abortion “stops a beating heart” are not only
scientifically inaccurate, but they are also morally unsound.

As high-risk pregnancy experts, we appreciate that seeing or hearing cardiac activity is a meaningful moment
for many people who are pregnant, and we share the joy of our patients who have a desired pregnancy. Two
critical questions arise, however: Should a flicker of cardiac activity on an ultrasound limit options available
to pregnant people, particularly those who desire or need to end their pregnancy? Furthermore, is a beating

8 | NLTI CLAT MOCK-4 – 26TH FEBRUARY 2023


heart either a necessary or sufficient standard upon which policy makers can decide the moral and legal status
of either a human or a fetus?

With decades of experience between us as physicians helping our patients grapple with the decisions a
complicated pregnancy can present, our answers to those questions are “no” and “no.”

Heartbeat bills are based on the work of an antiabortion, anti-LGBTQ, 2020-election-denier activist named
Janet Porter, who has been named the “godmother” or “crusader” of the heartbeat movement. Those behind
this movement believe that life begins at the onset of cardiac activity. The group Faith2Action first introduced
this antiabortion legislation in 2011 in Ohio, sending heart-shaped balloons to lawmakers and bringing
pregnant women to have the fetus “testify” via ultrasound exam. Since that time, multiple states have passed
bills based on this model legislation.

But this belief is scientifically incorrect.

The cells in the embryo that will become the heart begin developing three weeks after fertilization. Researchers
studying heart function have grown these cells, called myocytes, outside the body. With no heartlike structure,
and no blood cells to support them, myocytes can still contract rhythmically.

As early as five to six weeks after fertilization, before these structures resemble or function like a recognizable
heart, we can use ultrasound exams to see the rhythmic motion of the early heart cells as they respond to
electrical activity. It’s not until clinical week nine that we can see a recognizable four-chamber heart.

It’s these weeks of early development that are the target of antiabortion “heartbeat” laws, legislation that
misses a critical concept: a beating heart is not the only factor that determines legal status, moral status and
personhood.

But this doesn’t matter to antiabortion legislators, who place more value on the outcome of a pregnancy than
the person experiencing it. Speaking about the heartbeat bill he authored, Texas state senator Bryan Hughes
has said, “The heartbeat is the universal sign of life.” The legislative director of the group Texas Right To Life
has argued that “the heartbeat is a morally significant biological moment where we can detect whether
someone is alive or not.” These statements are wrong. Examples from other areas of medicine tell us that a
human heartbeat is neither a necessary nor a sufficient criterion upon which to base complex ethical decisions.

Having a heartbeat is not necessary to retain the full moral status of personhood. In some heart surgeries, for
example, the surgeon needs to chemically stop the heart to move and fix the tissue. In other situations, when
the heart can no longer pump blood, cardiac assist devices replace it. These people have no heartbeat, as we
would traditionally define it, yet no one would argue that they don’t retain their usual legal and moral status.
They have the same rights as someone whose heart is beating.
(Source: Han, C., & Heuser, C. C. (2023, January 23). Scientific American.)

21. What is the purpose of the argument presented in the passage?


a) To argue against the use of heart assist devices.
b) To show that heartbeat bills are scientifically and morally unsound.
c) To explain the development of the heart in an embryo.
d) To promote the use of heart surgeries.
9 | NLTI CLAT MOCK-4 – 26TH FEBRUARY 2023
22. Which of the following will be a strengthening argument for the aforementioned passage?
a) A study showing that cardiac activity can be seen in embryos as early as three weeks after
fertilization
b) A legal precedent establishing that a beating heart is sufficient evidence of personhood
c) A person declared brain-dead, they do not retain the same moral and legal status as other people,
despite having a heartbeat.
d) A statement from a medical expert supporting the belief that life begins at the onset of cardiac
activity

23. What is the tone of the passage regarding the heartbeat bill introduced by anti abortion
legislators?
a) Approving
b) Disapproving
c) Neutral
d) Ambivalent

24. What is the position of antiabortion legislators regarding the heartbeat bill?
a) That a beating heart is a necessary criterion to determine legal and moral status.
b) That a beating heart is not the only factor that determines personhood.
c) That a human heartbeat is neither a necessary nor sufficient criterion for complex ethical decisions.
d) That the development of the cardiovascular system is the universal sign of life.

25. The argument implies that a human heartbeat is


a) A necessary factor for complex ethical decisions.
b) A sufficient criterion for the legal and moral status of a person.
c) A universal sign of life.
d) Neither necessary nor sufficient for complex ethical decisions.

The idea of sleeping for an entire year first occurred to me while I was lying in bed, surrounded by dozens of
pillows, watching the snow fall outside my window. It was a grey day in early December, and the city felt
desolate and hushed, as if all its energy had been sucked away by the relentless blanket of snow.

I was feeling aimless and sad, and the idea of sleep seemed like a solution to my problem. I would sleep for a
year, and when I woke up, everything would be different. All my sorrows would be forgotten, and I would be
refreshed and renewed, ready to face the world again.

So I made a plan. I would quit my job, break up with my boyfriend, and isolate myself from the world for a
year. I would sleep as much as I could, and when I was awake, I would take a variety of drugs and prescription
medications to help me fall back to sleep.

I was fortunate enough to have a significant amount of savings, so I wasn't worried about money. I also had a
few friends who were more than willing to help me out, offering to bring me groceries and other necessities
while I slept.

10 | NLTI CLAT MOCK-4 – 26TH FEBRUARY 2023


I started my year of rest and relaxation in January, and at first, everything went according to plan. I slept for
hours on end, sometimes waking up briefly to eat or use the bathroom, but mostly I was lost in a deep,
dreamless sleep.

As the months went on, I started to experience strange and vivid dreams. Sometimes I dreamt of creatures that
were half-human, half-beast, roaming the city at night. Other times, I dreamt of being trapped in a maze, trying
to find my way out but always ending up back where I started.

At first, these dreams frightened me, but as I continued to sleep, I found myself becoming more and more
intrigued by them. They seemed to offer a glimpse into another world, a place that was both familiar and
strange.

As my year of rest and relaxation drew to a close, I started to feel a sense of unease. I was no longer sure that
sleep was the solution to my problems. Perhaps I had been foolish to believe that I could escape the world for
a year and return to it unchanged.

In the end, I woke up on the last day of December, feeling restless and uneasy. I was not the same person I
had been a year before, but I wasn't sure if I was better or worse for it. All I knew was that I was ready to face
the world again, with all its complexities and difficulties.

(Source: Excerpt from ‘Moshfegh, Ottessa. My Year of Rest and Relaxation. Penguin Books, 2019.’)

26. What can be inferred as the inspiration behind the narrator's idea to sleep for a year?
a) The narrator was feeling aimless and sad, and the idea of sleep seemed like a solution to their
problem.
b) The narrator wanted to escape from their job and their boyfriend.
c) The narrator had heard about someone who had successfully slept for a year.
d) The narrator was suffering from a medical condition that required extended periods of sleep.

27. According to the passage, what was the condition of the city when the narrator came up with the
idea of sleeping for a year?
a) The city was bustling with energy and activity.
b) The city was frigid and it was the holiday time.
c) The city was undergoing change, leaving it to be desolate.
d) The city was freezing and felt lifeless.

28. How did the narrator's plans change as they continued to sleep
a) The narrator realized that sleep was not a viable solution to their problems.
b) The narrator discovered that they were interested in the strange and vivid dreams they had while
sleeping.
c) They started taking prescription medications to help them fall asleep
d) The narrator discovered that they had wasted their time sleeping.

29. What conclusion did the narrator reach about their year of rest and relaxation?
a) The year of sleep was successful in helping the narrator forget their sorrows and become refreshed.
b) The narrator was disappointed in the results of their year-long sleep.
11 | NLTI CLAT MOCK-4 – 26TH FEBRUARY 2023
c) The narrator was not sure if they were better or worse off after the year of sleep.
d) The narrator felt that they were a different person after the year of sleep.

30. What is the general tone of the passage?


a) Optimistic and hopeful
b) Somber and melancholy
c) Suspenseful and mysterious
d) None of the above

12 | NLTI CLAT MOCK-4 – 26TH FEBRUARY 2023


GENERAL KNOWLEDGE SECTION

India on Thursday successfully tested a new variant of the BrahMos supersonic cruise missile from the
integrated test range at Chandipur off the coast of (1), the defence ministry said in a statement. The launch
was conducted by BrahMos Aerospace and the Defence Research and Development Organisation (DRDO).
According to the ministry statement, this variant of the missile was equipped with more indigenous
technologies than the previous ones. “The flight test is a major milestone for BrahMos programme. The
highly manoeuvrable missile cruised at supersonic speed for its maximum range and all mission objectives
were met,” the statement said. The BrahMos missile, an Indo-Russian joint venture, has a range of 290 km
and is the fastest cruise missile in the world. “The missile (tested on Thursday) was equipped with advanced
indigenous technologies and followed a modified optimal trajectory for enhanced efficiency and improved
performance. The missile with the modified control system has been fine tuned to achieve enhanced
capability,” the statement added.
Source: Hindustan Times

31.The sea-to-sea variant was test fired from INS Vishakhapatnam off the coast of
a) Visakhapatnam
b) Odisha
c) Karnataka
d) Kerala

32. BrahMos is a joint venture between the __________ and the NPOM of Russia
a) HAL
b) DRDO
c) Both a and b
d) None of the above

33. Which of the following is/or correct?


a) BrahMos is named on the rivers Brahmaputra and Moskva.
b) It operates on the "Fire and Focus" principle i.e it requires further guidance after launch
c) Brahmos is one of the fastest cruise missile currently operationally deployed with speed of Mach 2.8,
which is nearly 3 times more than the speed of sound.
d) It is a multiplatform missile i.e it can be launched from land, air, and sea and multi capability missile
with pinpoint accuracy that works in both day and night irrespective of the weather conditions.

34. In December 2021, the Defence minister of India laid the foundation for the BrahMos Aerospace
cruise missile manufacturing unit in
a) Lucknow
b) Bengaluru
c) Gandhinagar
d) Delhi

35. Who happens to be the first international customer for BrahMos?


a) Indonesia
b) Thailand
13 | NLTI CLAT MOCK-4 – 26TH FEBRUARY 2023
c) Philippines
d) China

The Ministry of Women and Child Development will celebrate the International Women’s Day week from
(I) to (II) March 2022 as an ‘Iconic Week’ as a part of ‘Azadi Ka Amrit Mahotsav’. As a part of the week-
long celebration for International Women’s Day, the Ministry will organize various events and social media
campaigns covering a variety of themes related to women’s safety and empowerment. The events are being
organised in partnership with national and international organisations and experts, with active participation
planned with women and children, as well as personnel who work directly for their protection and
empowerment.
The celebration will kick off tomorrow i.e., (X) March in collaboration with Bureau of Police Research and
Development to raise awareness on safety and security of women. On this day, the National Commission for
Protection of Child Rights (NCPCR) will also celebrate its Foundation Day. The focus of this event will be
children, wherein an exhibition by children will be on display and children will be taken on guided tour of
the Red Fort.

36. International Women’s Day Week is celebrated on which of these weeks?


a) 1st March -8th March
b) 9th March- 16th March
c) 6th March- 16th March
d) 7th March- 14th March

37. The United Nations celebrated its first official International Women’s Day on
a) 8th March 1975
b) 8th April 1975
c) 8th Febraury 1985
d) 8th June 1985

38. State which of these is/are correct?


a) In Mexico City, the UN World Conference on Women was held for the first time in June-July 1975.
As a result, the World Plan of Action was adopted as well as a declaration which led to the setting up
of monitoring mechanisms.
b) The United Nations declared the year 1975 as the ‘International Women’s Year.
c) in Russia in 1917, women held a protest known as the ‘Bread and Peace’ campaign on the last Sunday
in February. This demonstration started by women textile workers actually was the beginning of the
Russian Revolution.
d) All of the above

39. “Achieve gender equality and empower all women and girls” is construed under which SDG Goal?
a) Goal 3
b) Goal 4
c) Goal 5
d) Goal 6

40. India celebrates National Women’s Day on the occasion of Sarojini Naidu’s birthday on
a) 12th Febraury
14 | NLTI CLAT MOCK-4 – 26TH FEBRUARY 2023
b) 13th Febraury
c) 14th Febraury
d) 15th Febraury

World Heritage Day is observed every year on [X], to preserve the human heritage and recognize the efforts
of the organizations working for it. We all know that ancient buildings and monuments are an asset to us and
to the world. Therefore, World Heritage Day is a collective effort of the communities in the world to do
what is necessary.

A World Heritage Site is a place that is listed by the United Nations Educational Scientific and Cultural
Organization as of cultural importance. World Heritage Day gives us an opportunity to conserve and
preserve our heritage culture, which is of ancient significance. They have an outstanding universal value.

2.The World Heritage Day 2022 theme is [Y] to promote conservation research and employ sustainable
strategies to protect heritage sites. It is decided by the [Z] every year. [Z] wants people from various
religious backgrounds to set aside their differences and come forward and spread the message of solidarity.
The [Z] organization was established on the principles put down in the Venice Charter, also known as the
1964 [P]. During a seminar in Tunisia, the [Z] suggested celebrating World Heritage Day on [X], 1982.

Source: India Today


41. Which of the following will replace [X] in the first paragraph?
a) 16 April
b) 17 April
c) 18 April
d) 19 April

42. Who is the current head of the organisation labelled as [Z]?


a) Audrey Azoulay
b) Toshiyuki Kono
c) Markus Pleyer
d) Irina Bokova

43. Which of the following statement(s) is/are correct?


a) The theme ofWorld Heritage Day 2022 was ‘Heritages and Climate’.
b) Audrey Azoulay is the current Director-General who is heading UNESCO.
c) The Durga Puja celebrations in Howrah, West Bengal was recently declared Intangible Cultural
Heritage by UNESCO.
d) All of the above

44. Which were the first World Heritage Sites in India?


a) Agra Fort, Taj Mahal, Ajanta Caves, Ellora Caves
b) Red Fort, Raj Mahal, Ajanta Caves, Elephanta Caves
c) Red Fort, Taj Mahal, Ajanta Caves, Ellora Caves
d) Sun Temple Konark, Taj Mahal, Ajanta Caves, Elephanta Caves

45. Who is the current cabinet minister in ministry of culture in India?


a) Meenakshi Lekhi
15 | NLTI CLAT MOCK-4 – 26TH FEBRUARY 2023
b) G. Kishan Reddy
c) Maneka Gandhi
d) Sarbananda Sonowal

Union Minister of Commerce and Industry, Consumer Affairs, Food and Public Distribution and Textiles,
(1) said that fruitful discussions had been held over the course of the Indo-Pacific Economic Framework
(IPEF) Ministerial meeting to bring together a group of likeminded, rules-based, transparent countries with a
shared interest in an open Indo-Pacific region.
Highlighting that India had engaged very exhaustively in all the various streams of discussion, (1) said that
on three out of four pillars related to supply chains, tax and anti-corruption and clean energy, India was
comfortable with the outcome and text and had joined the declaration.
On one pillar, which deals primarily with trade, the Minister said, the contours of the framework -
particularly on commitments required on environment, labour, digital trade and public procurement — are
still emerging.
(1) also underscored that India was in the process of firming up our own digital framework and laws,
particularly regarding privacy and data and said that therefore India, while continuing to engage with the
trade track in the IPEF, will wait for the final contours to emerge. In the meantime, officials will be
participating in the discussions with an open mind and in the best interest of the people and businesses in
India, he added.
The Minister expressed confidence that together this group of (2) countries will define the rules of trade
among countries which believe in fair play, transparency and rules-based trading in the future.
Source: PIB

46. Who is the Minister of Consumer Affairs as substituted by (1)?


a) Prahlad Joshi
b) Piyush Goyal
c) Mahendra Nath Pandey
d) Sarbanda Sonowal

47.Which country initiated the Indo-Pacific Economic Framework?


a) UK
b) France
c) USA
d) Germany

48. How many members are there in the IPEF as substituted by (2)?
a) 14
b) 13
c) 12
d) 11

49. Which of the following is true about IPEF?


a) The IPEF has four pillars: Trade; supply chains; clean energy, decarbonisation and infrastructure; tax
and anti-corruption.
b) Except Cambodia, Laos and Myanmar, other Southeast Asian nations are a part of the IPEF.
c) The IPEF partners represent 40 per cent of global GDP and 28% of global goods and services trade.
16 | NLTI CLAT MOCK-4 – 26TH FEBRUARY 2023
d) All of the above.

50. When was the IPEF launched?


a) May 2022
b) May 2021
c) June 2021
d) June 2022

The Bay of Bengal Initiative for the Multi-Sectoral Technical and Economic Cooperation (BIMSTEC)
Forum recently celebrated its 25th anniversary on (1), coinciding with the signing of the (2) on the same
date in 1997.

In this landmark epoch of the Forum set up by the member countries, many discussions are happening about
the role played by the Forum and on whether an optimal situation has been reached in the 25 years of its
existence. Trade among BIMSTEC members accounts for about $40 billion, although it was earlier
estimated to be $250 billion for the last two years, much of the shortfall being due to the COVID-19
pandemic.
However, the larger points being discussed are about why BIMSTEC has not taken off despite mutual
goodwill among member countries and whether it has gone the SAARC way.
Although member states talk of their commitment to BIMSTEC, there is hardly any demonstrable action
towards meaningful regional cooperation or integration in the Bay of Bengal region. If member states are
serious about meaningful regional cooperation or integration, it has to necessarily involve the people of the
region. The movement of trade, goods, and people should be more free. But there is little movement on
initiatives towards advancing such programmes.
For a country like India, it becomes extremely important to provide proper leadership and solutions to all
problems in the region. That is why BIMSTEC’s main focus will always have to be uniting the Bay of
Bengal.
Source: The Quint

51. When did BIMSTEC celebrate its 25th anniversary, as substituted by (1) in the passage?
a) July 7th
b) June 6th
c) October 14th
d) September 6th

52. Which declaration led to the foundation of BIMSTEC in 1997?


a) Dhaka declaration
b) Delhi declaration
c) Bangkok declaration
d) Bandung declaration

53. Where is the BIMSTEC Technology Transfer Centre being established?


a) Bengaluru
b) Colombo
c) Dhaka
17 | NLTI CLAT MOCK-4 – 26TH FEBRUARY 2023
d) Kathmandu

54. Which of the following is not true about BIMSTEC?


a) The current chair of BIMSTEC is Sri Lanka.
b) It was established in 1997 but its first summit was held in 2004.
c) The permanent secretariat of BIMSTEC is located in Dhaka, Bangladesh.
d) None of the Above.

55. Who is the Secretary General of BIMSTEC?


a) Tenzin Lekphell
b) Lotay Tshering
c) Prayut Chan-o-cha
d) None of the Above.

(1) took oath of office as the 15th President of India on Monday and created history by being the first tribal
head of State and the second woman to occupy the country’s highest constitutional post. (1), who succeeds
Ram Nath Kovind, is India’s youngest President at 64 and the first to be born after Independence. She took
her oath of office in Hindi and in the name of God to "preserve, protect and defend the Constitution and the
law".
(1)’s speech in the Central Hall laid down her vision of sabka prayas (everyone's effort) and sabka kartavya
(everyone's duty) to fulfil the expectations of freedom fighters and citizens of independent India.
Starting out as BJP councillor in Odisha’s Rairangpur Notified Area Council in 1997, she became a minister
in Biju Janata Dal (BJD)-Bharatiya Janata Party (BJP) coalition government in the State between 2000 and
2004. Ms Murmu had contested the 2014 Assembly election from Rairangpur but lost to the BJD candidate.
A year later, in 2015, she was appointed Governor of Jharkhand and stayed in the post till 2021.
In honour of her remarkable journey through personal tragedies and triumphs, there were celebrations in
several places including her home state of Odisha as well as Jharkhand.
Source: The Hindu

56. Who became the 15th President of India, as substituted by (1) in the passage?
a) Pratibha Patil
b) Drupad Manakani
c) Draupadi Murmu
d) Durga Devi

57. Who was the first woman president of India?


a) Indira Gandhi
b) Pratibha Patil
c) Fathima Beevi
d) BV Nagarathna

58. Which Article of the Constitution provides the manner of election of the President?
a) Art. 51
b) Art. 53
c) Art. 55
18 | NLTI CLAT MOCK-4 – 26TH FEBRUARY 2023
d) Art. 57

59. Who is the current Vice-President of India


a) Bhairon Singh Shekhawat
b) Jagdeep Dhankar
c) M. Venkaiah Naidu
d) Md. Hamid Ansari

60. Which of the following is not true about the President's powers?
a) As per Article 53, the executive power of the country is vested in the president.
b) The president inaugurates parliament by addressing it after the general elections and also at the
beginning of the first session every year per Article 87(1).
c) The president appoints the Chief Justice of India and other judges on the advice of the chief justice.
d) All of the Above.

The (1) on Thursday easily passed a divisive resolution that proclaimed the safe access to abortion is a
human right — an effort to confront countries such as Poland and Malta, where women’s rights are being
restricted.
The resolution urges (1) countries to recognize that any interference with access to contraception, fertility
treatment, maternity care and abortion “constitute breaches of human rights.” It implores countries to
“condemn any attempt to limit access” to these services.
The measure passed with 378 voting in favor and 255 voting against.
The text — drafted by Predrag Fred Matić, a Croatian Socialist — is also critical of the so-called
“conscience clause,” which allows doctors to deny abortion access if they don’t believe in it. Withholding
abortions “on grounds of religion or conscience,” the text says, “endangers women’s lives and rights.”
The measure on Women’s Rights and Gender Equality overwhelmingly backed in May — primarily targets
countries such as Poland and Malta, which ban or restrict access to abortion. It also chides governments that
limited or revoked reproductive services during the coronavirus pandemic.
‘‘This vote marks a new era in 'X' and the first real resistance to a regressive agenda that has trampled on
women’s rights for years,” Matić said in a statement.

61. Who adopted a resolution on June 9 declaring that backsliding on the right to access safe and legal
abortion is a grave concern as substituted by (1)?
a) NATO Parliamentary Assembly
b) UN Parliamentary Assembly
c) EU Parliament
d) Parliamentary Assembly of Council of Europe

62. State which of the following is true


a) (1)'s lawmakers on Thursday passed a resolution calling for the bloc's 37 countries to ensure the right
to legal and safe abortion in a push to improve women's sexual and reproductive health.
b) The parliamentary resolution calls on member countries to step up regular health screenings, improve
access to contraception and fertility treatments, and combat menstrual poverty.
c) It calls for laws to be passed to safeguard the rights of intersex persons and demands better sexual
education for children.

19 | NLTI CLAT MOCK-4 – 26TH FEBRUARY 2023


d) Opponents said access to abortion would not improve women's health and that the right to life was
fundamental.

63. Who was elected as the President of (1) until 2024, with 458 votes, who happens to be the youngest
member to hold the office of President?
a) Roberta Metsola
b) Margrethe Vestager
c) Angela Merkel
d) Christine Lagarde

64. A recent landmark decision of the U.S Supreme Court held that the Constitution of the United
States does not confer a right to abortion
a) Obergefell v. Hodges
b) Roe v. Wade
c) Dobbs v. Jackson Women's Health Organisation
d) June Medical Services LLC v. Russo

65. The Supreme Court observed the artificial distinction between married and single woman for
aborting unwanted pregnancy is not constitutionally sustainable, which of the following is the bill
which seeks to provide necessary amendments to provide for abortion to women in India?
a) MTP Act, 1971
b) MTP (Amendment), 1971
c) MTP (Amendment) Bill, 2020
d) MTP Act, 2017

20 | NLTI CLAT MOCK-4 – 26TH FEBRUARY 2023


LEGAL REASONING SECTION
Section 300 of the IPC reads as follows: 300. Murder. —Except in the cases hereinafter excepted, culpable
homicide is murder, if the act by which the death is caused is done to cause death, or—(Secondly) —If it is
done to cause such bodily injury as the offender knows to be likely to cause the death of the person to whom
the harm is caused, which requires special knowledge or—(Thirdly) —If it is done to cause bodily injury to
any person and the bodily injury intended to be inflicted is sufficient in the ordinary course of nature to cause
death, or—(Fourthly) —If the person committing the act knows that it is so imminently dangerous that it must,
in all probability, cause death or such bodily injury as is likely to cause death, and commits such act without
any excuse for incurring the risk of causing death or such injury as aforesaid. Special knowledge is not
required.

• Causing death: There should be an intention of causing death


• Doing an act: There should be an intention to cause such bodily injury that is likely to cause death or
• The act must be done with the knowledge that the act is likely to cause the death of another.
Section 299 of IPC reads as follows: Culpable homicide — Whoever causes death by doing an act to cause
death, or intends to cause such bodily injury as is likely to cause death, or with the knowledge that he is likely
by such act to cause death, commits the offence of culpable homicide.
The difference between murder and culpable homicide is intention. If the intention is present the crime is
said to be committed under Section 300 of IPC. If the intention is absent, the crime is dealt with under
section 304 of IPC. The thin line is the intention behind the act. All murders are culpable homicide but vice-
versa is not true. Ever since the IPC was enacted, this distinction as to which case will fall under which
category is a perennial question with which courts are often confronted. On a plain reading of the relevant
provisions of the Code, it appears that the given cases can be conveniently classified into two categories but
when it comes to the actual application, the courts are often confronted with this dilemma. This confusion
often emerges when it is difficult to interpret from the evidence whether the intention was to cause merely
bodily injury which would not make out an offence of murder or there was a clear intention to kill the victim
making out a clear case of an offence of murder. The most confusing aspect is 'intention', which is to cause
death in both provisions. Hence, you have to consider the degree of the intention of offenders. If the person
is killed in cold blood or with planning then it is murder because the intention to kill is to a high degree and
not out of sudden rage or provocation. On other hand, if the victim is killed without pre-planning, in a
sudden fight or sudden anger because of somebody's provocation or instigation, then such a death is called
culpable homicide. Hence, whether the act done is culpable homicide or murder is a question of fact.

66. X was a student of A and had been studying with him for the past year. However, A always gave
him lesser marks as the answers were not up to the mark. X was enraged by this. He decided to go to
his house and talk some sense into him. Upon reaching A’s house, things heated up quickly and X
grabbed a knife and stabbed A in his chest. Decide.
a) The act committed by X is culpable homicide.
b) The act committed by X is Sec. 300 secondly.
c) The act committed by X is Sec. 300 secondly and fourthly.
d) The act committed by X is Sec. 300 thirdly.

67. Raghav and his girlfriend fought and argued a lot and they had been meaning to break up for a long
time now, both of them were looking for opportunities now. One day, things got very heated pretty
21 | NLTI CLAT MOCK-4 – 26TH FEBRUARY 2023
quickly and Raghav’s girlfriend pushed him a little hard. Raghav’s foot slipped and he hit his head on
the corner of a table and instantly died. Decide.
a) The girlfriend should be liable for murder.
b) The girlfriend should be liable for Sec. 300 fourthly.
c) The girlfriend should be liable for Sec. 299 due to lack of intention.
d) Insufficient facts.

68. Suppose in the above fact scenario, instead of Raghav falling to his death, his girlfriend had stabbed
him where he had a deep wound and a normal person would not have died with that stab. What would
be your answer now?
a) The girlfriend is liable for Sec. 300 firstly.
b) The girlfriend is liable for Sec. 300 secondly.
c) The girlfriend is liable for Sec. 300 thirdly.
d) The girlfriend is liable for Sec. 300 fourthly.

69. Kiara was admitted to a hospital in the emergency ward. She had been diagnosed with diabetes. A
nurse who worked there for ten years was given Kiara’s charge. One day, out of carelessness she
injected Kiara with the wrong injection and due to this Kiara passed away a few hours later.
a) The nurse is liable for culpable homicide.
b) The nurse is liable for murder as she had been careless.
c) The nurse is liable for murder as her intention can be assumed due to the act.
d) The nurse is liable for Sec. 300 fourthly.

70. Which of the following acts fall under Sec. 299 of the Indian Penal Code?
a) A driver driving at night and suddenly a person comes in front of the car and dies immediately.
b) A person shoots an animal but the bullet hit someone else.
c) An act of self-defence.
d) All of the above.

As general rule parties to a contract are having an intention towards the fulfilment of their part and in case of
a breach, the party breaching is liable to compensate for the same. But an exception to this rule is laid down
in Section 56 of the Indian contract act 1872. Section 56 deals with the doctrine of frustration as being acts
which cannot be performed. Under this doctrine, a promisor is relieved of any liability under a contract in
the event of a breach of contract and the contract will be deemed to be void. Section 56 is based on the
maxim “ les non-cogit ad impossible” which means that the law will not compel a man to do what he
cannot possibly perform.
The condition necessary for the application of Section 56

• There exists a valid and subsisting contract between the parties:- Existence of a valid contract
is the foremost condition for the application of Section 56. A valid contract includes a contract
entered in between competent persons and which is followed by some consideration.

• There must be some part of the contract which is yet to be performed:- Section 56 will have
applicability only if there is some part of the contract which is yet to be performed and without
performing it the ultimate purpose of the contract is not fulfilled.
22 | NLTI CLAT MOCK-4 – 26TH FEBRUARY 2023
• The contract after it is entered into becomes impossible to perform:- Another important condition
for the application of section 56 is that the contract after it has been entered into has become
impossible to perform and cannot be performed and therefore contract stands void.
• Initial impossibility:- The object of making any contract is that the parties to the contract would
perform their respective promises, and where the contract is impossible to perform the parties would
never enter into it. Initial impossibility deals with those cases where the contract was impossible to
perform from the very beginning.
• Subsequent impossibility:- It deals with cases where the contract was possible to perform when it
was entered but because of some event, the performance has become impossible or unlawful and
therefore it discharges the party from performing it.

71. Mr Sharma contracted with X for the delivery of 10 tonnes of wheat. X’s business was based in some
other country while Mr Sharma resided in India. When the contract was supposed to be initiated, there
developed war-like situations in X’s country, due to which X could not supply the wheat. Mr Sharma
sued for breach of contract. Decide.
a) Mr Sharma cannot sue for breach of contract.
b) Mr Sharma can sue for breach of contract.
c) The contract stands frustrated due to subsequent impossibility.
d) The contract stands frustrated due to initial impossibility.

72. A was a business tycoon and had dealt with multiple big-name companies. However, before he could
complete them, he suffered a heart attack and passed away. The companies decided to approach the
court.
a) The court should declare the contracts between A and the companies void.
b) The court should ask A’s company to fulfil its duties.
c) The court should compensate the plaintiffs.
d) Cannot be determined.

73. Renu was suggested by her neighbour that once he comes back from his trip, he would sell his
property to her. However, the neighbour decided to change his mind and when he came from his trip,
he decided to convey this to Renu. But Renu filed a complaint.
a) There is no contract.
b) There is frustration with the contract.
c) There is a breach of contract.
d) The neighbour has to go through with his promise.

74. Qala a singer, contracts a serious illness and is unable to fulfil a performance contract at the fixed
date for a performance at Golden Music Awards. Decide.
a) The contract is probably frustrated on the ground of the unavailability of the subject matter.
b) The contract is voidable.
c) The contract is probably frustrated on the ground of the unavailability of the agreed means of
performance.
d) Any alleged frustration is 'self-induced' and so the frustration doctrine will not apply.

23 | NLTI CLAT MOCK-4 – 26TH FEBRUARY 2023


75. Anand is unable to fulfil a contract for the sale of goods with Cavery Ltd because one of Anand's
suppliers' factories has been burned down by an accidental fire. Which one of the following most
accurately summarises the parties' legal position?
a) The contract between Anand and Cavery Ltd is probably frustrating because of the impracticability of
performance.
b) The contract between Anand and Cavery Ltd is probably frustrated by the physical destruction of the
subject matter.
c) The contract between Anand and Cavery Ltd is probably frustrating because the method of
performance has become impossible.
d) The contract between Anand and Cavery Ltd is probably frustrating based on the frustration of the
purpose of the contract.

“If any person commits suicide, whoever abets the commission of such suicide shall be punished with
imprisonment of either imprisonment for a term which may extend to ten years, and shall also be liable to
fine,” Section 306 reads. Generally, the fine is paid to the kin of the deceased.
The IPC also has a separate chapter on abetment and describes who is an abettor under Section 108. Abetment
is defined as instigating, engaging in a conspiracy or assisting in committing the offence.
Abetment of suicide is a serious offence that is tried in a Sessions court and is cognizable, non-bailable and
non-compoundable.
A cognizable offence is one in which a police officer can arrest without a warrant from a court. A non-bailable
offence means bail is granted to the accused at the discretion of the court and not as a matter of right.
A non-compoundable offence is one in which the case cannot be withdrawn by the complainant even when
the complainant and the accused have reached a compromise.
There are two primary ingredients of the crime of abetment of suicide. The first is a suicidal death. The second
ingredient is the intention of the accused to abet such suicide.
Legally, whether a death is a suicide or not is a determination of a fact, which means evidence has to be
evaluated to pronounce that death is a suicide. A determination of suicide is made when the deceased person
is understood to have known the probable consequence of their act of self-harm before proceeding to do it
intentionally.
Once such a determination is made, then the intention of the person accused of abetment of suicide is looked
into.
The only exception to this is the abetment of the suicide of a woman married for seven years or less. Through
an amendment in 1983 in the Code of Criminal Procedure, the law was changed to presume that the husband
is guilty if his wife commits suicide within seven years of the marriage. The amendment was made to curb
rising dowry deaths that were categorised as suicides.

76. Piyush and Suraj studied together in the same school. Both of them were really good friends,
however, Suraj always seemed to have been troubled by his family problems and suffered from mental
health issues. Once, Suraj told Piyush everything and said that he wanted to kill himself. Piyush did not
take this seriously and laughed it off, Suraj got very upset by this and the next day he killed himself.
Did Piyush abet Suraj’s suicide?
a) No, Piyush did not abet his suicide.
b) Yes, Piyush abet his suicide.
c) No, Piyush did not abet his suicide as there was no intention as well as instigation.
d) Yes, Piyush abetted Suraj’s suicide as there was an instigation.
24 | NLTI CLAT MOCK-4 – 26TH FEBRUARY 2023
77. Suppose in the above question, Suraj attempted to kill himself in front of Piyush and instead of
saving him, Piyush kept on shouting to him that he is a coward and he should kill himself, however, he
did not think Suraj would do that. Would your answer differ?
a) Yes, the answer would now differ.
b) Yes, the answer would now differ as Piyush had instigated Suraj.
c) No, the answer would not differ.
d) No, the answer would not differ as Piyush had no intention.

78. Raj was accused of abetting the suicide of his wife. The police came to his house and arrested him
without a warrant. Raj filed in court. Will his plea be sustained?
a) No, his plea will not be sustained.
b) Yes, his plea will be sustained.
c) Yes, his plea will be sustained as arresting without a warrant is against the law.
d) No, his plea will not be sustained as abetment to suicide is a cognizable offence.

79. Ishika was married to Aryan for 2 years. She was tortured daily by her husband about the dowry
given by her family during the time of their marriage, and how it was never enough. Getting tired of
this Ishika decided to end her life. Decide.
a) The court should presume Aryan’s intention as their marriage was still under the threshold of 7 years.
b) The court cannot presume his intention as it is not fair.
c) The court cannot presume his intention as there was no instigation.
d) The court should presume Aryan’s intention.

80. Yashi was a studious student and had always excelled in her studies. However, during COVID-19,
she could not attend a majority of her classes due to her online nature. This led to her missing out on a
lot of her studies. The teachers also kept on bothering her with schoolwork and exams. Eventually, she
failed all of her exams and committed suicide. Yashi’s parents filed a complaint against the teachers for
abetting the suicide of their daughter.
a) Yes, the teachers abetted the suicide of Yashi by being in a criminal conspiracy.
b) Yes, the teachers abetted the suicide of Yashi by instigating her towards taking her life.
c) No, the teachers did not abet the suicide of Yashi because there was no intention or instigation.
d) Insufficient facts.

Conspiracy means a combination of two or more persons for unlawful purposes. It is an agreement between
two or more persons to commit an illegal act. Section 120 of the I.P.C. defines criminal conspiracy as an
agreement of two or more persons to do or cause to be done-

1. An illegal act, or;


2. An act that is not illegal by illegal means.

Section 43 of the I.P.C. defines the term ‘illegal’ as everything that is an offence or is prohibited by law or
furnishes grounds for a civil action.
The Proviso attached to Section 120A provides that a mere agreement to commit an offence shall amount to
criminal conspiracy and no overt act or illegal omission is required to be proved. Such an overt act is
25 | NLTI CLAT MOCK-4 – 26TH FEBRUARY 2023
necessary only when the object of the conspiracy is the commission of an illegal act not amounting to an
offence. It is immaterial whether the illegal act is the ultimate object of such agreement or is merely
incidental to that object.
Two or more persons agree to do or cause to be done an illegal act or an act which is not illegal by illegal
means i.e. there must be at least 2 persons who conspire and there should be a meeting of minds. However,
a person may be indicted alone for the offence of criminal conspiracy if the other co-conspirators are
unknown, missing or dead.
Joint evil intent to do an illegal act or an act that is not illegal by illegal means is necessary. Ignorance of the
law is not an excuse.

81. Suresh decided to rob a bank because of his financial conditions. However, he knew he could not do
it alone and was also aware that it was an illegal act so no one would help him. He contacted one of his
old friends, Rakesh. Suresh asked him if he could use Rakesh’s house as a store for some of his cash as
his bank was not working. Rakesh decided to help his friend in need and agreed. When Suresh got
caught robbing and the cash was found at Rakesh’s house, the police arrested him too in a criminal
conspiracy.
a) Rakesh should also be arrested as he was involved in a crime.
b) Rakesh should not be arrested as there was no common intention to commit the crime.
c) Rakesh should not be arrested as he merely hid the cash and did not take part I. the actual robbing.
d) Rakesh should also be arrested as he acted in furtherance of an illegal act.

82. Radha, Sid and Varun were very close friends and studied in the same college. Sid’s mother was
very sick and he had to visit her. Sid could not do it as he had to travel to Canada and he did not have
a passport. Radha, Sid and Varun, all of them decided to gain Sid’s passport through illegal means. All
of them succeeded and Sid reached Canada through a legal visa issued on an illegal passport. But when
all of them got caught and charged under Section 120 of IPC, they pleaded that the act was a legal one.
a) Their plea should be upheld as the ultimate act was legal.
b) Their plea should not be upheld as the crime happened in Canada and it’s up to the Canadian
government.
c) Their plea should be upheld as they acted in good faith.
d) Their plea should not be upheld as there was an act was committed through illegal means.

83. Raj and Simran had a medical shop together and were in business for a long very time. One of their
customers asked them for a medicine that was not available with them at the moment. They ordered
the medicine for him but by the time the customer came, the sale of the medicine had been made illegal.
The police arrested Raj and Simran for criminal conspiracy.
a) Raj and Simran should be charged with criminal conspiracy.
b) Raj and Simran should not be charged with criminal conspiracy.
c) Raj and Simran should be charged with criminal conspiracy as ignorance of the law cannot be taken
as a defence.
d) Raj and Simran should not be charged with criminal conspiracy as when they had purchased the
medicines it was legal.

84. Manu was the daughter of a renowned politician but she was corrupt and an anti-nationalist. She
helped provide entry to four terrorists indirectly through her contacts. She never had any direct contact

26 | NLTI CLAT MOCK-4 – 26TH FEBRUARY 2023


with them. When the terrorists were tried in Court, they mentioned Manu’s connection too but she
denied all accusations saying she never helped them. Decide.
a) Manu cannot be charged with criminal conspiracy as there was no contact.
b) Manu cannot be charged with criminal conspiracy.
c) Manu can be charged with criminal conspiracy as there was a common intention.
d) Manu can be charged with criminal conspiracy

85. What if an act has been declared illegal and yet two or more people proceeded to continue with the
act with complete knowledge that that act is illegal? Would it still be covered in a criminal conspiracy?
a) Yes, it still would be covered under criminal conspiracy.
b) Yes, it still would be covered under criminal conspiracy as ignorance of the law is not an excuse.
c) No, it would not be covered under criminal conspiracy.
d) Cannot be determined.

A property can be divided into two parts; movable property and immovable property. Any offences
committed related to the property whether movable or immovable will be punishable under the
provisions of the Indian Penal Code, 1860. Provisions for offences relating to the property have been
enumerated under Section 378 – Section 460 of this code. Theft is defined under Section 378 of the
Indian Penal Code. Section 378 provides: Whoever, intending to take dishonestly any moveable
property out of the possession of any person without that person’s co nsent, moves that property to
such taking, is said to commit theft. There are five main ingredients of theft: Dishonest intention to
take property; Property should be moveable; The property must be taken out of possession of another
person; Property should be taken without the person’s consent, express or implied; Property should
be moved, to such taking. In the Rakesh v. State of NCT of Delhi case, the Hon’ble court held that
the mere intention of the offender to take property dishonestly out of the posse ssion of a person
without his consent is no offence. Until the property is removed, no offence has been committed.
The actual removal of property from another person’s possession is necessary, as mentioned
in Section 378 of IPC. In K.N. Mehra v. the State of Rajasthan case, the Supreme Court held that
proof of intention to cause permanent deprivation of property or to obtain wrongful gain is not
necessary for proving dishonest intention. The absence of a person’s consent to whom the property
belongs at the time of moving it and the presence of dishonest intention at the time of taking that
property are the essentials to commit theft.

86. A goes to B’s house and sees a diamond ring lying on the floor. Hides that ring at B’s place and
thinks whenever he will visit next time, He’ll take it. Now, choose the most appropriate option:
a) A is liable for theft as he hides the ring to take it next time.
b) A is not liable for theft as the ring has not been moved.
c) A is not liable for theft as the ring was lying on the floor.
d) None of the above.

87. Considering the fact situation in the first question, A hides the ring in a place where it is highly
improbable that it will ever be found by B and later A will take the ring from the hiding place when the
loss is forgotten. Now, choose the most appropriate option:
a) A is liable for theft when he takes the ring from B’s house
b) A is not liable for theft as the ring has not been moved.
27 | NLTI CLAT MOCK-4 – 26TH FEBRUARY 2023
c) A is liable for theft at the time of first moving the ring.
d) None of the above.

88. Considering the fact situation in the first question, if A had found the ring on the road outside B’s
house and not inside B’s house, he picks it up and keeps it in his pocket; then
a) A is liable for theft as he has taken the ring.
b) A is not liable for theft as the ring did not have anyone.
c) A is liable for theft as the ring was outside B’s house.
d) Either a or c

89. After reading the passage, what is theft of offence against?


a) Ownership
b) Possession
c) Both a and b
d) Neither a nor b

90. Statement I: Z goes to the library of his friend Y, with whom he is on good terms and borrows a
book without the consent of Y, with the intent of returning it afterwards. Z does not commit theft as
there is no dishonest intention attached to it.
Statement II: For the offence of theft, the property should be taken without the person’s consent,
express or implied.
a) Both the statements are individually true and statement II is the correct explanation of statement I
b) Both the statements are individually true and statement II is not the correct explanation of statement
I
c) Statement I is true but statement II is false
d) Statement I is false but statement II is true

Generally, a person is liable for his wrongful acts and one does not incur any liability f or the acts
done by others. In certain cases, like vicarious liability, the liability of one person for the act done
by another person may arise. The rule of vicarious liability imposes liability on one person for the
act done by another person. To hold a person liable for the act done by another person, there must
be a certain kind of relationship between the two persons and the wrongful act done should be, in a
certain way, connected with that relationship. Common examples of such relations include princi pal-
agent relationships, master-servant relationships and partners. However, a servant is different from
Independent Contractor. A servant is an agent who is subject to the control and supervision of his
employer regarding how the work is to be done, whereas an independent contractor is not subject to
any such control. He is his own master and exercises his discretion. Vicarious Liability of vehicle
owners: The liability pertains to the cases of an accident caused by mechanics, repairers or owners
of workshops during the test drive of the vehicles entrusted to them by the owners of the vehicles for
repairs. In B Govindarajulu v. MLA Govindaraja Mudaliar, a motor lorry was entrusted by its owner
for repair. While an employee of the workshop was driving it the lorry met with an accident. It was
held by the Madras High Court that the lorry owner was not vicariously liable because the owner of
the workshop was an independent contractor and not a servant of the owner. Vicarious (Tortious)
Liability of State: The liability of the State for the tortious acts of its servant is known as the tortious

28 | NLTI CLAT MOCK-4 – 26TH FEBRUARY 2023


liability of the State. The liability of the State for the torts committed by its servants is based on the
following three principles:
o Respondent superior i.e. let the principal be held liable.
o Qui facit per alium facit per se i.e. he who acts through another does the act himself.
o Socialisation of compensation.

91. Keeping in view the principles set out in the above passage, which of the following statements is
correct?
a) An innocent principal is not liable for the fraud of his agent.
b) An innocent principal is liable for the fraud of his agent.
c) An innocent agent is liable for all the frauds of his principal.
d) None of the above.

92. Mr B was under the employment of Mr A as a driver of his car. Mr A wants to go to the airport to
catch a flight. He asks Mr B to drop him at the airport. While driving to the airport Mr B negligently
knocks down Mr X, who was crossing the road. In a suit by Mr X against Mr B.
a) Mr A is liable as the negligence was done by Mr B during the course of his employment
b) Mr A is not liable as the negligence was done by Mr B.
c) No one is liable as Mr X was crossing the road and he should have been careful.
d) None of the above.

93. Considering the facts of the question above, Mr A hired a taxi to drive him to the airport instead of
taking his car. Mr C was driving the taxi. While driving to the airport Mr C negligently knocks down
Mr X, who was crossing the road. In a suit by Mr X against Mr C and Mr A, who will be liable
a) Both Mr C and Mr A will be liable as Mr C was hired by Mr A.
b) Only Mr C will be liable as he was the driver.
c) Only Mr A will be liable as he was the one who hired Mr C.
d) Only Mr C will be liable as Mr C was not the servant of Mr A but only an independent contractor.

94. Some children(ages 10-11) of a school had gone on a picnic under the charge of two teachers of the
school. Before lunch, some of the children decided to take bath in the near river on the bank on which
they had camped. While swimming, two of the children drowned. It was found that the teachers in
charge were taking lunch at that time. In a suit for compensation:
a) No one would be liable because the accident was due to the recklessness on part of the children
themselves.
b) The headmaster would be liable for the negligence of the teachers as he had permitted sightseeing.
c) The school administration would be liable for breach of duty to take care of teachers.
d) All of the above.

95. A has an account in Punjab National Bank. B is an employee of the same bank and is living in a
house next to A. A gives some money to B to deposit in the bank account of A. B misappropriates the
money. In this case, the bank is:
a) Liable for money misappropriated by B as he is its employee.
b) Liable as the master is vicariously liable for the torts committed by its servants.
c) Not liable as the money was not received by B during the course of his employment.
d) Liable as B was in employment by the Bank when the money was given to him.
29 | NLTI CLAT MOCK-4 – 26TH FEBRUARY 2023
Interpretation of Statutes means the process by the help of which the courts interpret or expl ain and
apply legislation. There are certain rules of Interpretation that the court applies from time to time
depending on the circumstance. One of the primary and cardinal rules of Interpretation is the Literal
rule. According to, to this rule of Interpretation the Courts interpret the statutes literally and
grammatically to give the words their ordinary and natural meaning. Therefore, if after Interpretation
the meaning of the text is clear and ambiguous then effect shall be given to the provisions of the
statute. And thus, it is not important to look into the legislative intent. The judges thus, have the duty
to interpret the words as it is given without any addition or deletion to the same. In simple words, it
can be said that the Literal Rule of Interpretation means what the law says instead of what the law
means. It is one of the safest rules of interpretation of statutes as the very intention of the legislature
is deduced from the words and the language used therein. In the latest landmark judgement ti tled
Commercial Taxes Officer, Circle-B, Bharatpur vs M/s Bhagat Singh, the Apex Court in the exercise
of its extraordinary appellate jurisdiction held that a statute must be interpreted in a just, reasonable
and sensible manner. Most remarkably, the Bench then makes it crystal clear in para 18 which forms
the cornerstone of this notable judgment that, “The Court must interpret a statute in a manner which
is just, reasonable and sensible. If the grammatical construction leads to some absurdity or some
repugnancy or inconsistency with the legislative intent, as may be deduced by reading the provisions
of the statute as a whole, the grammatical construction may be departed from to avoid anomaly,
absurdity or inconsistency. This rule has certain advantages such as it helps the layman understand
the statute, the intention of the Legislature is expressed simply and clearly, the supremacy of the
Parliament in the administration of justice etc. While it has certain demerits such as it might lead to
unreasonable decisions, its application in all situations is not possible etc.

96. What does the Interpretation of Statutes means?


a) The words are to be given their plain meaning
b) The intention of the Legislature must be determined
c) The explanation and application of any legislation by the Court
d) All the above

97. What does the Literal Rule of Interpretation mean?


a) The ordinary or natural meaning of the text of the statutes
b) The literal and grammatical rule of interpretation
c) What the law means
d) Both (a) & (b)

98. In the Commercial Taxes Officer, Circle-B, Bharatpur vs M/s Bhagat Singh it was held that the
statutes must be interpreted in a just, reasonable and sensible manner unless___________
a) There arises some absurdity with the legislative intent
b) The parties prove that other rules should be adopted
c) Both (a) & (b)
d) None of the above

99. At the entrance of the Zoo it’s written: “Dogs and Cats are not allowed”. Is the restriction only apply
to Dogs and Cats, and therefore, Cows can be brought to the Zoo?
a) Yes, it is allowed as the restriction is only for Dogs and Cats
30 | NLTI CLAT MOCK-4 – 26TH FEBRUARY 2023
b) No, it is not allowed as the restriction is on the category of Pet animals
c) No, it is not allowed as the restriction on animals
d) No, it is not allowed as Zoo is not a place for an animal walk

100. Saurabh was caught along with the counterfeit currency “dollars” and he was charged under
Indian Penal Code for possessing counterfeit currency. Can he be charged?
a) Yes, he can be charged for possessing counterfeit currency as currency is not limited to Indian
Currency only.
b) Yes, he can be charged as he is counterfeiting.
c) No, he can’t be charged for possessing counterfeit currency as dollars are not the legal currency in
India.
d) No, he cannot be charged as he is not dealing in dollars.

Double jeopardy is a procedural defence that forbids a defendant from being tried twice for the same
crime for the same set of facts. It refers to the concept that it is wrong for a man to be subjected more
than once to the danger of being punished for an offence. The roots of the doctrine again st double
jeopardy can be found in the well-established maxim of the English Common law, Nemo debit bis
vexari, meaning that a man must not be put twice in peril for the same offence. The Double Jeopardy
Clause encompasses four distinct prohibitions: subsequent prosecution after acquittal, subsequent
prosecution after conviction; subsequent prosecution after certain mistrials, and multiple
punishments in the same indictment. Article 20 of the Constitution of India protects respect of
conviction for offences and Article 20(2) against double jeopardy. It has been enshrined as a part of
the Fundamental Rights by the fathers of our Constitution. Accordingly, no person can be prosecuted
and punished for the same offence more than once. The provision apotheosize s the principle that a
person cannot be tried twice for the same offence by an equally competent court. In India, partial
protection against double jeopardy (Autrefois convict) is a Fundamental Right guaranteed under
Article 20 (2) of the Constitution of India. It states that “No person shall be prosecuted and punished
for the same offence more than once”. However it does not extend to autrefois acquit, and so if a
person is ‘acquitted’ of a crime can be retried. The protection against autrefois acquit is a statutory
right in our country and not a fundamental right.

101. Based on the above passage, what does double jeopardy mean?
a) Trying two persons jointly for the same offence
b) Trying the same person for two offences at two different times.
c) Trying a person for two offences committed by him in one incident.
d) Putting the same person on trial twice for the same offence.

102. Article 20 of the Constitution of India protects against?


a) Civil Liability
b) Criminal Liability
c) Both civil and criminal liability
d) None of the above.

103. A meets Z on the high roads, shows a pistol, and demands Z’s purse. Z in consequence surrenders
his purse. Here A has extorted the purse from Z by putting him in fear of instant hurt and being at the
time of committing the extortion in his presence. A has therefore committed robbery. An action was
31 | NLTI CLAT MOCK-4 – 26TH FEBRUARY 2023
brought against him under the said provision for the blog. Now, choose the most appropriate option
amongst the following.
a) A is liable for the offence of extortion and robbery as both of these offences were executed.
b) A is liable only for the offence of robbery as robbery is a more serious offence than extortion.
c) A is liable only for the offence of robbery as both offences are deemed the same and A can be
punished only once.
d) Both B and C.

104. Considering the fact situation in the above question, A hears a police siren and runs from the scene,
dropping the purse on the road. A is caught in the nearby colony. However, he is acquitted due to a lack
of evidence. Can A be tried again for theft?
a) Yes
b) No
c) Can’t say
d) None of the above.

105. Considering the fact situation in Question 103, A after hearing the police siren fires the pistol and
runs from the scene. Identify for which of the offences A will be liable.
a) Both robbery and murder as both of these are different offences.
b) Only robbery as the murder was not planned.
c) Only murder as murder is a more grave offence than robbery.
d) None of the above.

32 | NLTI CLAT MOCK-4 – 26TH FEBRUARY 2023


LOGICAL REASONING
As anyone who excavates fossils will tell you, finding evidence of Ice Age children is difficult. It’s not just
that their small, fragile bones are hard to locate. To understand why we forget about them in our
reconstructions of prehistory, we also need to consider our modern assumptions about children. Why do we
imagine them as ‘naive’ figures ‘free of responsibility’? Why do we assume that children couldn’t contribute
meaningfully to society? Researchers who make these assumptions about children in the present are less
likely to seek evidence that things were different in the past.
But using new techniques, and with different assumptions, the children of the Ice Age are being given a
voice. And what they’re saying is surprising: they’re telling us different stories, not only about the roles they
played in the past, but also about the evolution of human culture itself.
Human bones are fragile things, but some are more fragile than others. The larger, denser bones of adults
tend to be better preserved in the archaeological record than those of children, whose bones are more like a
bird’s than an elephant’s: they are smaller, more porous and less mineralised, lack tensile and compressive
strength, and may not be fully fused to their shafts (in the case of long bones). These skeletons are more
vulnerable to both sedimentary pressure (when buried underground) and erosion from acidic soil and
biodegrading organic matter. This is one of the main reasons why telling the stories of prehistoric children
has been so difficult.
But they’re not only poorly preserved. The small size of some remains means they can be easily missed. I
experienced this when I worked at an archaeological site called Drimolen in South Africa, 40 km north of
Johannesburg. Archaeologists working here have dated the site to between 1.5 and 2 million years old, and
have uncovered the remains of more than 80 individuals who are early members of the Homo genus and the
Paranthropus robustus species, another ancient human lineage. Close to 50 per cent of the uncovered
remains were identified as children under 10 years old – far more than has been recovered from similar sites
in the region (but a figure that is more in line with estimates of Ice Age demographics). One reason for this
difference is the rigorous screening protocol followed by the project’s team. Every cubic centimeter of earth
that is excavated at Drimolen is dry-sieved to pick out larger pieces, then placed on medium- and fine-mesh
screens and cleaned with running water. The remaining sediment is spread out on a table, ready to be sorted.
During my time at Drimolen, I spent countless hours searching for even the tiniest human tooth, hidden
among stones and the skeletal remains of rodents and other small animals.

106. Why is it difficult to find evidence of Ice Age children?


a) They lived in isolated communities.
b) They had little impact on society.
c) Their bones are fragile and easily missed.
d) They lived too long ago to be preserved.

107. What is one reason why the bones of children are more vulnerable to erosion and sedimentary
pressure?
a) Children's bones are more dense than adult bones.
b) Children's bones are larger than adult bones.
c) Children's bones are less mineralized than adult bones.
d) Children's bones are less strong than adult bones.

108. What was the author's experience at the Drimolen archaeological site?
a) The author found many Ice Age children's bones.

33 | NLTI CLAT MOCK-4 – 26TH FEBRUARY 2023


b) The author found very few Ice Age children's bones.
c) The author found many adult bones, but no children's bones.
d) The author spent a lot of time searching for small human teeth.

109. What makes the discovery of Ice Age children at Drimolen site significant?
a) The remains were well-preserved.
b) The remains were dated to a recent time period.
c) The remains were of a different species than expected.
d) The remains were of a higher proportion of children than other sites.

110. What are the new techniques revealing about the Ice Age children?
a) They lived in isolated communities.
b) They had little impact on society.
c) They played different roles in the past.
d) They did not contribute meaningfully to society.

We tend to avoid straight talk about death and killing. Still, the phrase the vet had chosen to broach the
subject of euthanising our cat struck me as especially evasive. When I was a child in the 1970s, pets were
‘put down’. Today, even this phrase sounds harsh, despite its obliqueness. It then became more common to
talk of putting the animal to sleep, as though death were as gentle and pleasant as a good nap. ‘Saying
goodbye’ takes euphemism to new extremes, removing any sense of agency, as though their parting were an
event that simply happened.
We soon did ‘say goodbye’ to Pixel and it hurt to do so. Our news elicited stories from others of deep
mourning for their pets. One said he was ‘in pieces for days’, a phrase that resonated with my partner, who
cried more than I had ever seen her do before. There is nothing phoney about our professed love for our pets.
Although it felt more like bereavement for a person than the loss of a thing, the death of a pet isn’t exactly
like either because our relationships with our animals is unlike any other. We are close to our animals while
at the same time being worlds apart, unable to truly understand what it is like to be a member of their
species. Our ties with them can be deep and involve reciprocity but they are also full of asymmetries. We
value their lives greatly but not usually in anything like the same way as we do those of our fellow humans.
Those who genuinely put their pets on the same level as other people are rightly considered eccentric, if not
monstrous.
Thinking of our pets as family occludes the deep tensions that are present throughout our lives with them.
Reflecting on Pixel’s passing brought these to the fore. It made it clearer than ever that, although we can
have incredibly strong bonds with animals, they are not our friends. We can value their lives intensely but
when their number is up, no matter how hard it hits us, I don’t think it is the awful event the death of a
fellow human is. Thinking about my relationship with Pixel shed light on our relationship to nature more
widely, as well as the difficulty of seeing it for what it is, in all its splendour and cruelty.

111. What is the author's opinion on the phrase "saying goodbye" in reference to euthanizing a pet?
a) It is too extreme and removes the sense of agency.
b) It is appropriate and describes the event accurately.
c) It is too harsh and should not be used.
d) It is gentle and makes the process easier.

34 | NLTI CLAT MOCK-4 – 26TH FEBRUARY 2023


112. How does the author feel about the death of their pet, Pixel?
a) They were not affected by it.
b) They were relieved it happened.
c) They were devastated by it.
d) They were indifferent to it.

113. What is the author's opinion on people who consider their pets to be on the same level as
humans?
a) They are misguided.
b) They are normal.
c) They are eccentric.
d) They are monstrous.

114. According to the author, what is the relationship between humans and their pets like?
a) It is a close and reciprocal bond.
b) It is distant and unreciprocal.
c) It is a friendship.
d) It is a family relationship.

115. How does the author's relationship with their pet, Pixel, relate to their relationship with nature
more widely?
a) It has no relationship.
b) It highlights the difficulty of seeing nature for what it is.
c) It shows that humans can have close bonds with nature.
d) It emphasizes the importance of considering pets as family.

Cleopatra Selene was born in 40 BCE, in the wake of her parents’ renowned and fabled encounter in the city
of Tarsus, so memorably described by Antony’s biographer Plutarch, invoked by William Shakespeare, and
painted by Sir Lawrence Alma-Tadema. This meant that she was around 10 years old when the civil war
between her father and his fellow triumvir Gaius Julius Caesar Octavianus (known today as Octavian, he
would go on to become the first Roman emperor Augustus) culminated in Octavian’s victory over Antony
and Cleopatra at the Battle of Actium in 31 BCE, and his conquest and annexation of Egypt the following
year. After her parents’ suicides, she was taken back to Rome by Octavian and spent the remainder of her
childhood in the household of his sister Octavia, who also happened to be her father’s ex-wife and the
mother of two of his daughters, Cleopatra Selene’s half-sisters.
While Octavian’s biographer Suetonius claimed that he was a kindly father figure and reared her as carefully
as if she was his own flesh and blood, there was undoubtedly a political dimension to this decision, and it
was probably his foremost concern. Retaining control of Cleopatra Selene meant that any potential threat to
Rome’s power over Egypt was neutralised before it could gain any momentum. It would be impossible for
anyone to attempt to lay claim to Egypt while the last scion of the Ptolemaic dynasty, the true heir to the
throne, was alive and well. But that is not to say that Octavian’s tenure as Cleopatra Selene’s guardian was
entirely benign.
Octavian forced Cleopatra Selene to participate in the events of the third day of his Triple Triumph in the
summer of 29 BCE: this third and final day commemorated his conquest of Egypt and, in the absence of
their mother, Cleopatra Selene and her twin brother Alexander Helios walked alongside an effigy of
35 | NLTI CLAT MOCK-4 – 26TH FEBRUARY 2023
Cleopatra entwined with the snakes that had supposedly ended her life. Cleopatra Selene was dressed as the
Moon and Alexander Helios as the Sun in reference to the celestial and divine names that Antony had
bestowed upon them in addition to their more prosaic Ptolemaic names that they shared with numerous other
members of the dynasty. This was undoubtedly Octavian’s attempt to ensure that the crowd of thousands
lining the processional route would be certain to recognise them. Luckily for them, unlike other enemies of
Rome such as Vercingetorix of Gaul, their participation in a military triumph did not culminate in their ritual
execution.

116. What was the reason for Cleopatra Selene being taken to Rome after the defeat of her father
Antony?
a) To train her to be the future ruler of Egypt
b) To prevent her from making a claim to the throne of Egypt
c) To educate her in Roman culture
d) To honor her parents' legacy

117. Who was the biographer of Octavian who claimed that he was a kindly father figure to Cleopatra
Selene?
a) Antony
b) Plutarch
c) Suetonius
d) William Shakespeare

118. What was the purpose of Cleopatra Selene and her brother Alexander Helios participating in the
third day of Octavian's Triple Triumph?
a) To commemorate their father's victory over Octavian
b) To commemorate Octavian's conquest of Egypt
c) To show their loyalty to Octavian
d) To honor their mother

119. What was the outcome of Cleopatra Selene and her brother Alexander Helios' participation in
the military triumph?
a) They were executed
b) They were honored
c) They were exiled
d) They were not harmed

120. What was the significance of Cleopatra Selene being dressed as the Moon and Alexander Helios
as the Sun in Octavian's triumph?
a) It symbolized their celestial and divine names given by Antony
b) It symbolized their power as members of the Ptolemaic dynasty
c) It was a way for Octavian to control their appearance
d) It was a way for the crowd to recognize them.

Games – from 5,000-year-old boardgames to last year’s video-game blockbusters – are miniature
laboratories for playing with possibility. Because play never happens in isolation (we are always playing
36 | NLTI CLAT MOCK-4 – 26TH FEBRUARY 2023
with something, after all), games can be a way of deepening our connections to each other. Play puts us in a
strange dance of cooperation and competition, where we come to know others and ourselves more deeply.
A game is a conflict. That’s true whether two teams are facing off for a basketball game, a handful of friends
are role-playing a fantasy boardgame around a kitchen table, or you are absorbed in your latest smartphone
puzzle addiction. Every game pits players with or against each other in a system of conflict. This ‘conflict’
might sound negative, a kind of antagonistic competition. In fact, the conflict in games is always
collaborative in some way. This is because everyone participating agrees, voluntarily, to take part in the
game together. If we’re being forced to play, it’s not really play. We all decide to spend the next couple of
minutes – or hours, or weeks – within the space of play, and together keep the struggle of the game going.
Even when we play alone, we are in a sense collaborating as part of a community of players who are all
playing the same game.
The ongoing struggle of games weaves energy and engagement into the experience. When we play, we are
taking part in productive conflict, joyful conflict, meaningful conflict. Conflict is part of the dramatic
machinery of a game that grips our minds and emotions. As every storyteller knows, there is no drama
without conflict. Conflict is the spark that helps games catch fire in us.
Part of the fear that games sometimes evoke – for example, the fear of violent video games leading to real
violence – comes from a misunderstanding of our relationship to the conflict inherent in games. When we
play a game, we do not become confused about whether or not the game is real. In fact, we are able to lose
ourselves in play because (paradoxically!) we know that games are artificial. The conflict in games is like
two actors fighting on a stage: artificial, theatrical combat. The audience members watching from their seats
don’t rush up on stage to intervene and stop the fight. Instead, they sit in the theatre and suspend their
disbelief. They can be gripped by the drama of the fight, yet at the same time know that it’s artificial.

121. What is the main idea of the passage?


a) Games are a way of deepening our connections with each other through collaboration and conflict.
b) Games evoke fear due to a misunderstanding of conflict.
c) The struggle in games creates energy and engagement.
d) Conflict is essential for all games.

122. How does the author view the conflict in games?


a) Negative and antagonistic
b) Collaborative and voluntary
c) Confusing and real
d) Artificial and theatrical

123. What is the relationship between play and conflict in games according to the author?
a) Conflict is a negative aspect of play.
b) Conflict is essential for productive play.
c) Play is the opposite of conflict.
d) Conflict is accidental in play.

124. What is the author's view on the fear of violent video games leading to real violence?
a) It is a valid concern.
b) It is due to a misunderstanding of the relationship between games and conflict.
c) It is not justified.
d) It is a consequence of artificial conflict in games.
37 | NLTI CLAT MOCK-4 – 26TH FEBRUARY 2023
125. How does the author compare the conflict in games to a theatrical fight?
a) The conflict in games is more real than a theatrical fight.
b) The conflict in games is like a theatrical fight in that it is artificial.
c) The conflict in games is less dramatic than a theatrical fight.
d) The conflict in games has no comparison to a theatrical fight.

Archaeology as a profession faces two major problems. First, it is the poorest of the poor. Only paltry sums
are available for excavating and even less is available for publishing the results and preserving the sites once
excavated. Yet archaeologists deal with priceless objects every day. Second, there is the problem of illegal
excavation, resulting in museum-quality pieces being sold to the highest bidder.
I would like to make an outrageous suggestion that would at one stroke provide funds for archaeology and
reduce the amount of illegal digging. I would propose that scientific archeological expeditions and
governmental authorities sell excavated artifacts on the open market. Such sales would provide substantial
funds for the excavation and preservation of archaeological sites and the publication of results. At the same
time, they would break the illegal excavator’s grip on the market, thereby decreasing the inducement to
engage in illegal activities.
You might object that professionals excavate to acquire knowledge, not money. Moreover, ancient artifacts
are part of our global cultural heritage, which should be available for all to appreciate, not sold to the highest
bidder. I agree. Sell nothing that has unique artistic merit or scientific value. But, you might reply everything
that comes out of the ground has scientific value. Here we part company. Theoretically, you may be correct
in claiming that every artifact has potential scientific value. Practically, you are wrong.
I refer to the thousands of pottery vessels and ancient lamps that are essentially duplicates of one another. In
one small excavation in Cyprus, archaeologists recently uncovered 2,000 virtually indistinguishable small
jugs in a single courtyard. Even precious royal seal impressions known as l’melekh handles have been found
in abundance—more than 4,000 examples so far.
The basements of museums are simply not large enough to store the artifacts that are likely to be discovered
in the future. There is not enough money even to catalogue the finds; as a result, they cannot be found again
and become as inaccessible as if they had never been discovered. Indeed, with the help of a computer, sold
artifacts could be more accessible than are the pieces stored in bulging museum basements. Prior to sale,
each could be photographed and the list of the purchasers could be maintained on the computer. A purchaser
could even be required to agree to return the piece if it should become needed for scientific purposes.
It would be unrealistic to suggest that illegal digging would stop if artifacts were sold on the open market.
But the demand for the clandestine product would be substantially reduced. Who would want an unmarked
pot when another was available whose provenance was known, and that was dated stratigraphically by the
professional archaeologist who excavated it?

126. According to the passage, what are the two major problems faced by the profession of
archaeology?
a) Scarcity of funds and illegal excavation
b) Lack of appreciation and poor storage
c) Inadequate funding and insufficient storage space
d) Unauthorized digging and limited access to artifacts

127. What is the author's proposal to provide funds for archaeology and reduce illegal excavation?
38 | NLTI CLAT MOCK-4 – 26TH FEBRUARY 2023
a) Increase government funding for excavations
b) Sell excavated artifacts on the open market
c) Improve the accessibility of artifacts stored in museums
d) Increase public awareness of the importance of archaeology

128. What does the author believe about the scientific value of every artifact that comes out of the
ground?
a) They all have the same value
b) They have potential scientific value but not practical value
c) They have scientific value but not artistic value
d) They have both scientific and artistic value

129. According to the passage, what is the problem with the storage of artifacts in museums?
a) There is a shortage of storage space
b) There is a shortage of money to catalogue the finds
c) The finds are difficult to access
d) All of the above

130. What is the author's belief about the impact of selling artifacts on the open market on illegal
excavation?
a) It will increase the amount of illegal digging
b) It will have no impact on illegal excavation
c) It will substantially reduce the demand for illegally excavated artifacts
d) It will increase public awareness of the importance of preserving archaeological sites.

It is an odd but indisputable fact that the seventeenth-century English women who are generally regarded as
among the forerunners of modern feminism are almost all identified with the Royalist side in the conflict
between Royalists and Parliamentarians known as the English Civil Wars. Since Royalist ideology is often
associated with the radical patriarchalism of seventeenth century political theorist Robert Filmer—a
patriarchalism that equates family and kingdom and asserts the divinely ordained absolute power of the king
and, by analogy, of the male head of the household—historians have been understandably puzzled by the
fact that Royalist women wrote the earliest extended criticisms of the absolute subordination of women in
marriage and the earliest systematic assertions of women’s rational and moral equality with men. Some
historians have questioned the facile equation of Royalist ideology with Filmerian patriarchalism; and
indeed, there may have been no consistent differences between Royalists and Parliamentarians on issues of
family organization and women’s political rights, but in that case one would expect early feminists to be
equally divided between the two sides.

Catherine Gallagher argues that Royalism engendered feminism because the ideology of absolute monarchy
provided a transition to an ideology of the absolute self. She cites the example of the notoriously eccentric
author Margaret Cavendish (1626–1673), duchess of Newcastle. Cavendish claimed to be as ambitious as
any woman could be, but knowing that as a woman she was excluded from the pursuit of power in the real
world, she resolved to be mistress of her own world, the “immaterial world” that any person can create
within her own mind—and, as a writer, on paper. In proclaiming what she called her “singularity,”
Cavendish insisted that she was a self-sufficient being within her mental empire, the center of her own
subjective universe rather than a satellite orbiting a dominant male planet. In justifying this absolute
39 | NLTI CLAT MOCK-4 – 26TH FEBRUARY 2023
singularity, Cavendish repeatedly invoked the model of the absolute monarch, a figure that became a
metaphor for the self-enclosed, autonomous nature of the individual person. Cavendish’s successors among
early feminists retained her notion of woman’s sovereign self, but they also sought to break free from the
complete political and social isolation that her absolute singularity entailed.

131. What is the main issue addressed in the passage?


a) The difference between Royalists and Parliamentarians in seventeenth-century England.
b) The relationship between Royalism and feminism.
c) The effect of seventeenth-century English women on modern feminism.
d) The relationship between Royalist ideology and patriarchalism.

132. What was the Royalist ideology associated with according to the passage?
a) Equality between men and women.
b) The radical patriarchalism of Robert Filmer.
c) The pursuit of power by women.
d) The eccentricity of Margaret Cavendish.

133. How did Margaret Cavendish attempt to assert her ambitions as a woman in seventeenth-century
England?
a) By claiming to be as ambitious as any man.
b) By seeking to be a satellite orbiting a dominant male planet.
c) By becoming the mistress of her own mental empire.
d) By breaking free from political and social isolation.

134. What was Catherine Gallagher's argument about the relationship between Royalism and
feminism?
a) Royalism and feminism were inconsistent with each other.
b) Royalism engendered feminism through the ideology of absolute monarchy.
c) Royalism and feminism had no connection.
d) Royalist women were equally divided between Royalism and Parliamentarianism.

135. According to the passage, what did Margaret Cavendish's successors among early feminists
retain from her beliefs?
a) The notion of a woman's sovereign self.
b) The complete political and social isolation.
c) The model of the absolute monarch.
d) The relationship between Royalist ideology and patriarchalism.

40 | NLTI CLAT MOCK-4 – 26TH FEBRUARY 2023


QUANTITATIVE APTITUDE
Directions(136-140): Study the following information carefully and answer the questions given beside.

NLTI, after great success in the recent examination, has planned a trip to reward the employees for their
hard work and dedication. The company has 4200 employees. 72% of the employees registered for the trip.
The ratio of male and female employees registered for the trip is 7: 5. Out of them, 75% of the employees
have chosen the company transport services for the trip and the rest decided to use their own vehicle. On the
trip day, 25% of those who have chosen for company transport, went on the trip by their own vehicles for
some reasons. The ratio of male and female employees who came by their own transport is 5: 4. Initially, the
company had booked 57 buses with 40 seat capacity in each.

136. What is the number of female employees who registered for the trip?
a) 1080
b) 1260
c) 1320
d) 1180

137. Find the difference between the total number of male and female employees who came to the trip
by their own transport?
a) 155
b) 135
c) 147
d) 138

138. How many buses are actually used for the trip?
a) 43
b) 40
c) 45
d) 38

139. The number of employees who went to the trip by their own transport in spite of registering for
the company transport is what percent of the number of employees who did not register for the trip?
a) 35.23%
b) 50.67%
c) 56.43%
d) 48.21%

140. Find the ratio of the number of female employees who registered for the trip and number of
female employees who came to the trip by their own transport?
a) 9 : 5
b) 11 : 6
c) 15 : 7
d) 13 : 9

Directions (Q. 141 – 145) Study the following information carefully and answer the given questions:

41 | NLTI CLAT MOCK-4 – 26TH FEBRUARY 2023


NLSIU Law Review Journal recently updated their website, due to which they are able to check the
geographical statistics related to the viewers of the journal. On analysing this information, Annie, an Observer
in the journal, collated the following data.
Following pie chart shows the distribution of the total number of readers of the NLSIU Law Review Journal
in the year 2015 and 2016 among different states.

141. If the total number of NLSIU Law review journal readers of the state AP in the year 2015 and
2016 is 57200 and 45600 respectively, then find the difference between the total number of readers of
the state Kerala in the year 2015 to that of 2016?
a) 3200
b) 2700
c) 2400
d) 3500

42 | NLTI CLAT MOCK-4 – 26TH FEBRUARY 2023


142. If the ratio of total number of NLSIU Law review journal readers of the state AP in the year
2016 to that of the state MP in the year 2015 is 4 : 5, then find the ratio of total number of readers of
all the given states together in the year 2016 to that in the year 2015?
a) 72 : 95
b) 67 : 88
c) 53 : 92
d) 61 : 76

143. If the total number of NLSIU Law review journal readers of all the given states together in the
year 2015 and 2016 is 24 lakhs and 28 lakhs respectively, then find the total number of readers of the
state MP in the year 2015 and 2016 together?
a) 11.5 lakhs
b) 14.6 lakhs
c) 17.3 lakhs
d) 10.2 lakhs

144. Total number of NLSIU Law review journal readers of state AP in the year 2016 is
approximately what percentage of total number of readers of state MP in the year 2015? (Assume that
the total number of readers for both years is the same.)
a) 94 %
b) 106 %
c) 88 %
d) 72 %

145. If the total number of NLSIU Law review journal readers of state Kerala in the year 2015 is
84000 and the total number of readers of state MP in the year 2016 is 52500, then the ratio of total
number of readers of state AP in the year 2015 to that of total number of readers of state Kerala in
the year 2016?
a) 157 : 143
b) 181 : 158
c) 165 : 134
d) 154 : 135

Directions (Q. 146 – 150): Study the following information carefully and answer the questions given
below:
Sardar Anish, an excellent accountant, was requested by Mr Agarwal to analyse the economic productivity
of the two shops owned by him. Depending on the result, his infusion of capital would be determined. The
following data is what Sardar Anish calculated.

43 | NLTI CLAT MOCK-4 – 26TH FEBRUARY 2023


146. Total number of items sold by P in the week is what percent of the total number of items sold by
Q in the week?
a) 110.8%
b) 101.8%
c) 102.7%
d) 103.4%

147. Number of items sold by shop R in Wednesday and Thursday together is what percent more than
the number of items sold by shop S in Thursday and Friday together?
a) 21.11%
b) 32.12%
c) 31.11%
d) 18.25%

148. Find the respective ratio of number of items sold by shop R in Sunday, Monday and Tuesday
together and the number of items sold by shop S in Wednesday, Thursday and Friday together.
a) 23:21

44 | NLTI CLAT MOCK-4 – 26TH FEBRUARY 2023


b) 21:23
c) 22:25
d) 25:22

149. Find the total number of items sold by shop R in the week.
a) 3083
b) 3830
c) 3803
d) 3038

150. Find the difference between the total number of items sold by shop Q and shop S in the week.
a) 45
b) 29
c) 20
d) 30

45 | NLTI CLAT MOCK-4 – 26TH FEBRUARY 2023

You might also like